Quiz Discussion

\(6\frac{5}{6} \times 5\frac{1}{3} \times 17\frac{2}{3} \times 4\frac{1}{2} = ?\)

 

Course Name: Quantitative Aptitude

  • 1]

    \(112\frac{1}{3}\)

  • 2]

    \(116\frac{2}{3}\)

  • 3]

    240

  • 4]

    663

  • 5]

    None of these

Solution
No Solution Present Yet

Top 5 Similar Quiz - Based On AI&ML

Quiz Recommendation System API Link - https://fresherbell-quiz-api.herokuapp.com/fresherbell_quiz_api

# Quiz
1
Discuss

Simplify : \({ \text{8}}\frac{1}{2} - \left[ {3\frac{1}{4} + \left\{ {1\frac{1}{4} - \frac{1}{2}\left( {1\frac{1}{2} - \frac{1}{3} - \frac{1}{6}} \right)} \right\}} \right]\)

 

  • 1]

    \(4\frac{1}{2}\)

  • 2]

    \(4\frac{1}{6}\)

  • 3]

    \(9\frac{1}{2}\)

  • 4]

    \(\frac{2}{9}\)

Solution
2
Discuss

If 12 + 22 + 32 + . . . . . + p2 = \(\left[ {\frac{{{ \text{p}}\left( {{ \text{p}} + 1} \right)\left( {2{ \text{p}} + 1} \right)}}{6}} \right]{ \text{,}}\)     then 12 + 32 + 52 + . . . . . + 172 is = ?

 

  • 1] 969
  • 2] 1785
  • 3] 980
  • 4] 1700
Solution
3
Discuss

98643 – 21748 = 51212 + ?

  • 1]

    24383

  • 2]

    24713

  • 3]

    25683

  • 4]

    26243

Solution
4
Discuss

\(\sqrt {\frac{{{{\left( {0.03} \right)}^2} + {{\left( {0.21} \right)}^2} + {{\left( {0.065} \right)}^2}}}{{{{\left( {0.003} \right)}^2} + {{\left( {0.021} \right)}^2} + {{\left( {0.0065} \right)}^2}}}}\)

 

  • 1] 0.1
  • 2] 10
  • 3] 102
  • 4] 103
Solution
5
Discuss

If \(x = \sqrt 3 { \text{ + }}\sqrt 2 { \text{,}}   \) then the value of \({x^3} - \frac{1}{{{x^3}}}\)  is?

 

  • 1]

    \(10\sqrt 2 \)

  • 2]

    \(14\sqrt 2\)

  • 3]

    \(22\sqrt 2\)

  • 4]

    \(8\sqrt 2\)

Solution
6
Discuss

The least number that must be subtracted from 63522 to make the result a perfect square is = ?

  • 1] 18
  • 2] 20
  • 3] 24
  • 4] 30
Solution
7
Discuss

If 37/13 =   where x,y,z are natural member , then find x+y+z are:-

  • 1]

    6

  • 2]

    7

  • 3]

    8

  • 4]

    9

Solution
8
Discuss

\(\frac{3}{7}{ \text{of }} 455 + \frac{5}{8}{ \text{of}}456 = ?\)

 

  • 1] 448
  • 2] 464
  • 3] 476
  • 4] 480
  • 5] None of these
Solution
9
Discuss

Simplification of \(\frac{{{{\left( {3.4567} \right)}^2} - {{\left( {3.4533} \right)}^2}}}{{0.0034}} = ?\)

 

  • 1] 6.91
  • 2] 7
  • 3] 6.81
  • 4] 7.1
Solution
10
Discuss

Simplify : \(\frac{{ - \frac{1}{2} - \frac{2}{3} + \frac{4}{5} - \frac{1}{3} + \frac{1}{5} + \frac{3}{4}}}{{\frac{1}{2} + \frac{2}{3} - \frac{4}{3} + \frac{1}{3} - \frac{1}{5} - \frac{4}{5}}}\)

 

  • 1]

    -3/10

  • 2]

    -10/3

  • 3]

    -2

  • 4]

    1

Solution
# Quiz